28 svar
227 visningar
ilovechocolate är nöjd med hjälpen
ilovechocolate 664
Postad: 6 nov 2021 16:06

Bestämma I, e och Q

Hur löser jag denna uppgift? Förstår inte ens vart jag ska börja…

Ture 9897 – Livehjälpare
Postad: 6 nov 2021 16:42

eftersom du vet potentialen i punkten p kan du beräkna strömmen i det horisontella 100 ohms motståndet.

Sen är det strömdelning mellan de två lodräta motstånden

Kan det vara en bra start?

Alternativt använder du kirschoffs lagar i de två slingorna.

ilovechocolate 664
Postad: 6 nov 2021 17:21 Redigerad: 6 nov 2021 17:36

Okej, så över resistorn 100 får kan U=44,72… V vilket i sin tur ger I=0,4472… A

Hur ska jag göra då det är strömdelning? Bör jag slå ihop båda resistorerna och skriva om det till Rp=(400•100)/(400+100)=80 ohm? 

Försökte använda kirchhoffs spänningslag, men kom typ ingenstans 

Ture 9897 – Livehjälpare
Postad: 6 nov 2021 17:55

Var kom 44,72 från?

jag tolkar symbolen längst ut till vänster, det lilla strecket som en jordpunkt.

ilovechocolate 664
Postad: 6 nov 2021 17:59

Ja juste, glömde bort den! Då får jag ut strömmen genom P=R•I^2 vilket ger I=0.4472….

Programmeraren 3387
Postad: 6 nov 2021 18:09

P i figuren är bara en viss punkt i kretsen, den kunde hetat Stefan eller vad som helst. Den har ingen med effekt att göra.
I punkten P är potentialen 20V
I jordpunkten är potentialen 0V
Spänning är samma sak som potentialskillnad. T ex ett 9V-batteri har potentialskillnaden (spänningen) 9V mellan polerna.

ilovechocolate 664
Postad: 6 nov 2021 18:12

Juste vad dumt, blandade ihop p:et med effekt. Då blir det ju I=R/P=5A

PATENTERAMERA 5477
Postad: 6 nov 2021 18:16

P betecknar en punkt i kretsen, inte effekten. vP är elektriska potentialen i denna punkt.

Använd ohms lag. Spänningen över det horisontella motståndet är hur mycket? Vad blir då strömmen genom det horisontella motståndet?

ilovechocolate 664
Postad: 6 nov 2021 18:21
PATENTERAMERA skrev:

P betecknar en punkt i kretsen, inte effekten. vP är elektriska potentialen i denna punkt.

Använd ohms lag. Spänningen över det horisontella motståndet är hur mycket? Vad blir då strömmen genom det horisontella motståndet?

Insåg det, räknade ju om det till I=R/P=5A

Programmeraren 3387
Postad: 6 nov 2021 18:23

Nu blev det ett slarvfel.
U=IR --> I=U/R = (20-0)/100=0,2

ilovechocolate 664
Postad: 6 nov 2021 18:25 Redigerad: 6 nov 2021 18:34
Programmeraren skrev:

Nu blev det ett slarvfel.
U=IR --> I=U/R = (20-0)/100=0,2

Hoppsan, det går verkligen inte bra för mig. De här kretsarna har förstört huvudet ☹️ Okej, så strömmen blir alltså 0,2 A. Åt vilket håll går strömmen från resistorn, dvs medurs eller moturs? hur går jag vidare härifrån? Ska jag göra samma sak på den andra 100-resistorn?

Programmeraren 3387
Postad: 6 nov 2021 18:55

Strömmen går från högre potential till lägre potential (minnesregel: som vatten från en högre höjd till en lägre).

Nu när du vet strömmen genom den högra R=100, hur fördelar den sig genom de nedre R=400 och R=100?

ilovechocolate 664
Postad: 6 nov 2021 19:05

Så det sker en strömfördelning? Såhär kanske I=14001400+1100×i, där i är strömmen från 100-resistorn

Jag har bara väldigt svårt att se allt framför mig.

PATENTERAMERA 5477
Postad: 6 nov 2021 19:14

Det är dumt att använda i för strömmen genom det horisontella motståndet, eftersom det är strömmen genom 400 ohms motståndet, vilket även är en storhet som skall vara med i svaret.


Tillägg: 6 nov 2021 19:17

Och kolla upp strömgreningsformeln här.

ilovechocolate 664
Postad: 6 nov 2021 19:23 Redigerad: 6 nov 2021 19:47

Okej, men ser ju ut som att jag gjorde rätt. Kan skriva såhär istället då I=14001400+1100×Io=0,04 A

PATENTERAMERA 5477
Postad: 6 nov 2021 19:53

Ja det ser rätt ut. Vad blir nästa steg?

ilovechocolate 664
Postad: 6 nov 2021 19:58

Såg att e=Io•Rtot=0,2•180=36V och vidare Vq=RI=82•0,2=16,4 V

PATENTERAMERA 5477
Postad: 6 nov 2021 20:22

Går det någon ström genom 82Ω motståndet?

ilovechocolate 664
Postad: 6 nov 2021 20:32 Redigerad: 6 nov 2021 20:32

Nej. Då den delen inte är sluten… men åh, hur gör jag då? 

PATENTERAMERA 5477
Postad: 6 nov 2021 20:37

Om det inte går någon ström genom motståndet, hur stor är spänningen över motståndet?

ilovechocolate 664
Postad: 6 nov 2021 20:40

Spänningen är väll 0 V då? 

PATENTERAMERA 5477
Postad: 6 nov 2021 21:24

Precis, det betyder att det inte är någon potentialskillnad mellan punkten Q och den horisontella ledaren längst ner i figuren. Så Q har samma potential som ledaren längst ner i figuren. Vilken potential har denna ledare?

ilovechocolate 664
Postad: 6 nov 2021 21:43 Redigerad: 6 nov 2021 21:45

Har den  alltså samma potential som e då, dvs 36 v? Det kan väll inte stämma…🤔

Programmeraren 3387
Postad: 6 nov 2021 22:08

Potential är inte samma sak som spänning. Spänning är skillnad i potential. Jag förklarade lite om det i början av tråden.

Antingen vet man en punkts potential, i detta fall vp=20V, eller så väljer man en punkt och definierar den till 0V.  Om du sen går längs en ledare är potentialen oförändrad så länge du inte kommer till en komponent. Passerar du en spänningskälla från minus till plus ökar potentialen med batteriets spänning (från till plus till minus så minskar potential med spänningen). Passerar du ett motstånd i strömriktningen så minskar potential med I*R. I uppgiften går ingen ström genom 82 ohms-motståndet och därför är det samma potential på båda sidor om det.

Du vet potentialen i P. Om du går nedåt genom e, som du redan beräknat spänningen över, vad är då potentialen nedanför e?

PATENTERAMERA 5477
Postad: 6 nov 2021 23:58

Spänningen e är potentialskillnaden mellan punkten P och punkten A - se figur. Dvs

e = vP - vA. Vilket ger vA = vP - e.

Sedan hade vi redan konstaterat att vQ = vA.

ilovechocolate 664
Postad: 7 nov 2021 00:16

Jaha okej! Och då Va=Vp-e=20-36=-16 V

ilovechocolate 664
Postad: 7 nov 2021 00:17
Programmeraren skrev:

Potential är inte samma sak som spänning. Spänning är skillnad i potential. Jag förklarade lite om det i början av tråden.

Antingen vet man en punkts potential, i detta fall vp=20V, eller så väljer man en punkt och definierar den till 0V.  Om du sen går längs en ledare är potentialen oförändrad så länge du inte kommer till en komponent. Passerar du en spänningskälla från minus till plus ökar potentialen med batteriets spänning (från till plus till minus så minskar potential med spänningen). Passerar du ett motstånd i strömriktningen så minskar potential med I*R. I uppgiften går ingen ström genom 82 ohms-motståndet och därför är det samma potential på båda sidor om det.

Du vet potentialen i P. Om du går nedåt genom e, som du redan beräknat spänningen över, vad är då potentialen nedanför e?

Tack för förklaringen. Min hjärna vill så hemskt gärna alltid blanda ihop potential och spänning, då båda har enheten volt 

Programmeraren 3387
Postad: 7 nov 2021 07:59

Att "potentialvandra" genom kretsen är i princip samma sak som Kirchhoffs spänningslag, vandrar man igenom hela slingan måste potentialen bli det värde man började på. Finns ingen potential angiven är det enklast att tilldela 0V till den punkt man tror har lägst potential, ofta på minussidan av en spänningskälla. Om någon annan punkt senare visar sig ha lägre potential spelar ingen roll, då kommer den att få negativ potential. Ofta användbart för att hitta samband i kretsar.

ilovechocolate 664
Postad: 7 nov 2021 13:02

Tack, ska tänka på det 😃

Svara Avbryt
Close